Difference between revisions of "2005 AMC 10A Problems/Problem 17"

(Solution 2)
(Solution 2)
Line 11: Line 11:
 
Since the middle term in an arithmetic sequence is the average of all the terms in the sequence, the middle number is <math>\frac{60}{5}=\boxed{\textbf{(D) }12}</math>
 
Since the middle term in an arithmetic sequence is the average of all the terms in the sequence, the middle number is <math>\frac{60}{5}=\boxed{\textbf{(D) }12}</math>
  
==Solution 2==
+
==Solution 2 (Doesn't assume a solution exists) ==
We know that the smallest number in this sequence must be <math>3 + 5 = 8</math>, and the biggest number must be <math>7 + 9 = 16</math>. Since there are <math>5</math> terms in this sequence, we know that <math>8 + 4d = 16</math>, or that <math>d = 2</math>. Thus, we know that the middle term must be <math>8 + 2 \cdot 2 = \boxed{12}.</math> This doesn’t necessarily work since not every single number will be added to every other number, but it is a good way to start trying during the test.  
+
We know that the smallest number in the arithmetic sequence must be <math>\geq 3 + 5 = 8</math>, and the largest number must be <math>\leq 7 + 9 = 16</math>.  
~yk2007 (Daniel K.)~sravya_m18
+
 
 +
Since there are <math>5</math> terms in this sequence, the common difference <math>d \leq (16 - 8)/(5-1) = 2</math>.
 +
 
 +
Since <math>d</math> is an integer (difference of sums of integers), and since exactly 2 of the sums must be odd, <math>d</math> must be odd. Therefore, <math>d=1</math>.
 +
 
 +
The middle term must have the majority parity, so it must be odd.
 +
The 2 terms adjacent to the middle are odd, <math>6+a</math> and <math>6+b</math>. <math>a-b = 2d = 2</math>. <math>a</math> and <math>b</math> can't be the smallest (or largest) 2 odd numbers, because that would make it impossible to construct the smallest (or largest) sum from one of the remaining two numbers and one of the odd numbers. Therefore, <math>\{a,b\} = \{5,7\}. The middle sum must then be </math>(6+5) + 1 = (6+7)-1 = \boxed{12}<math>. The remaining edges are </math>\{9,3\}<math> (because {5,7} can't be an edge, as that would make a triangle with 6), </math>\{3,7\}<math>, and </math>\{5,9\}$.
 +
 
 +
~oinava
  
 
== Video Solution by OmegaLearn ==
 
== Video Solution by OmegaLearn ==

Revision as of 00:35, 9 July 2024

Problem

In the five-sided star shown, the letters $A, B, C, D,$ and $E$ are replaced by the numbers $3, 5, 6, 7,$ and $9$, although not necessarily in this order. The sums of the numbers at the ends of the line segments $AB$, $BC$, $CD$, $DE$, and $EA$ form an arithmetic sequence, although not necessarily in that order. What is the middle term of the arithmetic sequence?

2005amc10a17.gif

$\textbf{(A) } 9\qquad \textbf{(B) } 10\qquad \textbf{(C) } 11\qquad \textbf{(D) } 12\qquad \textbf{(E) } 13$

Solution 1

Each corner $(A,B,C,D,E)$ goes to two sides/numbers. ($A$ goes to $AE$ and $AB$, $D$ goes to $DC$ and $DE$). The sum of every term is equal to $2(3+5+6+7+9)=60$

Since the middle term in an arithmetic sequence is the average of all the terms in the sequence, the middle number is $\frac{60}{5}=\boxed{\textbf{(D) }12}$

Solution 2 (Doesn't assume a solution exists)

We know that the smallest number in the arithmetic sequence must be $\geq 3 + 5 = 8$, and the largest number must be $\leq 7 + 9 = 16$.

Since there are $5$ terms in this sequence, the common difference $d \leq (16 - 8)/(5-1) = 2$.

Since $d$ is an integer (difference of sums of integers), and since exactly 2 of the sums must be odd, $d$ must be odd. Therefore, $d=1$.

The middle term must have the majority parity, so it must be odd. The 2 terms adjacent to the middle are odd, $6+a$ and $6+b$. $a-b = 2d = 2$. $a$ and $b$ can't be the smallest (or largest) 2 odd numbers, because that would make it impossible to construct the smallest (or largest) sum from one of the remaining two numbers and one of the odd numbers. Therefore, $\{a,b\} = \{5,7\}. The middle sum must then be$(6+5) + 1 = (6+7)-1 = \boxed{12}$. The remaining edges are$\{9,3\}$(because {5,7} can't be an edge, as that would make a triangle with 6),$\{3,7\}$, and$\{5,9\}$.

~oinava

Video Solution by OmegaLearn

https://youtu.be/tKsYSBdeVuw?t=544

~ pi_is_3.14

See Also

2005 AMC 10A (ProblemsAnswer KeyResources)
Preceded by
Problem 16
Followed by
Problem 18
1 2 3 4 5 6 7 8 9 10 11 12 13 14 15 16 17 18 19 20 21 22 23 24 25
All AMC 10 Problems and Solutions

The problems on this page are copyrighted by the Mathematical Association of America's American Mathematics Competitions. AMC logo.png